Stumped by an Integral: Can it be Solved with Elementary Functions?

Click For Summary
The integral ∫(arctan{x})/(x(x²+1)) dx is challenging and likely cannot be expressed in terms of elementary functions. Attempts to solve it using substitutions such as u=tan{x} or u=arctan{x} lead to complex results, including terms involving the logarithm and polylogarithms. Tools like Mathematica and TI-89 confirm that the integral does not yield a classical solution. Despite extensive efforts and various substitutions, no elementary function solution has been found. The consensus is that this integral likely does not have a solution in elementary terms.
silver-rose
Messages
47
Reaction score
0

Homework Statement



\int\frac{\arctan{x}dx}{x(x^2+1)}

I've been thinking over this for the past few days...I'm still stuck though
Can this integral even be expressed with elementary functions?


Homework Equations


N/A


The Attempt at a Solution


use the substitution u=tan{x}, and then, use integration by parts.
However I end up with \int\left|ln(cosx)\right|, as a term, which I cannot manage to integrate.
 
Last edited:
Physics news on Phys.org
Hint: Use u=arctan(x). What is du?
 
I think this integral does not have a "classical" primitive. After the substitution and partial integration you end up with:

I=\frac{1}{2}\frac{arctan^2(x)}{x^2}+\frac{1}{2}\int \frac{u^2}{sin^2(u)}du

The remaining integral is not an elementary function, according to "the integrator" of mathematica.

@silver-rose: What is expected as a result? A classical function or an advanced one?
 
I agree. I can't find an elementary function despite a few pages of calculations and random substitutions. Maybe it's because I'm dumb or something. Anyone else had better luck here?

Tried it out at http://integrals.wolfram.com/

The answer was given in some weird notation involving something called a polylogarithm. What's that?
 
Last edited:
Sorry my bad, I thought the OP had got it wrong in substituting u=tan(x) instead of arctan(x), and didn't check further. Since the mathematica integrator doesn't report a solution in terms of elementary functions, it is highly unlikely that there actually exists one.
 
yea that's what i think so too..

I've spent days on this integral, basically trying tons and tons of substitutions.

ti-89 can't do it, and mathematica gives a non-elementary answer.

Thanks anyways guys.
 
Question: A clock's minute hand has length 4 and its hour hand has length 3. What is the distance between the tips at the moment when it is increasing most rapidly?(Putnam Exam Question) Answer: Making assumption that both the hands moves at constant angular velocities, the answer is ## \sqrt{7} .## But don't you think this assumption is somewhat doubtful and wrong?

Similar threads

Replies
3
Views
2K
  • · Replies 6 ·
Replies
6
Views
2K
  • · Replies 14 ·
Replies
14
Views
3K
  • · Replies 1 ·
Replies
1
Views
1K
  • · Replies 22 ·
Replies
22
Views
3K
  • · Replies 105 ·
4
Replies
105
Views
6K
  • · Replies 8 ·
Replies
8
Views
2K
  • · Replies 9 ·
Replies
9
Views
3K
Replies
3
Views
1K
Replies
19
Views
2K